Équivalent d’une suite du dimanche après-midi

etanche
Modifié (November 2021) dans Analyse
Bonjour

1/ $a_1=1 ,\ a_{n+1} = \dfrac{ \sqrt{n} }{1+a_n}$. Donner un équivalent puis un développement à deux termes.
2/ $a_1=1 ,\ a_{n+1} = \dfrac{ n^{\alpha} }{1+a_n}$, où $\alpha \neq 1/2$ est un réel .
Mêmes questions

Merci.

Réponses

  • Bonjour,

    1) Conjecture:
    $$a_n \underset{n\to +\infty}{=} n^{1/4}-\displaystyle\frac{1}{2}+o(1).

    $$ (Correction (signe $+$ remplacé par signe $-$) effectuée après le premier message de Frédéric Bosio).
  • 1/ Je cherche $b_n>0$ tel que $b_{n}=\frac{\sqrt{n}}{1+b_{n}}$. C'est $b_{n}=\frac{1}{2}(-1+\sqrt{1+4\sqrt{n}})$.
    J'ai l'impression que pour $n \ge 5$ on a : $b_{n-1}<a_{n}<b_{n}$, et que la suite $a_n$ est croissante pour $n \ge 4$.
  • Pas plutôt $n^{1/4} - 1/2 $ ?
  • Oui, tu as bien raison: je vais modifier mon message initial.
  • Pour la première, je parierais pour \[u_n=n^{1/4}-\frac12+\frac18n^{-1/4}+\frac18n^{-3/4}+o(n^{-3/4}).\]Le dernier terme n'est pas très étayé.
  • En surfant sur la suggestion de Chaurien et le développement de Mathcoss si $a_{n+1}=\frac{f(n)}{1+a_{n}}$ avec $a_1>0$ et $f(n)>0$ croissante (conditions à préciser peut-être) a-t-on ?
    $$a_{n}=f(n)^{1/2}-\frac{1}{2}+\frac{1}{8}f(n)^{-1/2}+O\left(f(n)^{-3/2}\right).
    $$ Quelques essais numériques le laissent penser. La forme fraction continue $$a_n=\frac{f(n)}{1+\frac{f(n-1)}{1+\frac{f(n-2)}{1+\frac{...}{f(1)}}}}$$ ne semble pas apporter grand chose.
  • Je fais une tentative d'avancée avec ma fonction croissante $f>0$ précédente. Si on peut montrer comme l'a remarqué Chaurien avec le premier exemple que $a_n$ est croissante à partir d'un certain rang comme $a_{n+1}a_{n}+a_{n+1}=f(n)$ on aurait pour $n$ suffisamment grand $f(n)>a_{n}^{2}+a_{n}$. D'un autre côté $f(n-1)=a_{n}a_{n-1}+a_{n}<a_{n}^{2}+a_{n}$ donc
    $$
    f(n-1)<a_{n}^{2}+a_{n}<f(n)
    \\
    f(n-1)<\left(a_{n}+\frac{1}{2}\right)^{2}-\frac{1}{4}<f(n)
    \\
    \left(f(n-1)+\frac{1}{4}\right)^{1/2}-\frac{1}{2}<a_{n}<\left(f(n)+\frac{1}{4}\right)^{1/2}-\frac{1}{2}.
    $$

    Ajout: en prenant $f(n)=n^{1/2}$ on a
    $$\left(f(n-1)+\frac{1}{4}\right)^{1/2}-\frac{1}{2}=n^{1/4}-\frac{1}{2}+\frac{1}{8}n^{-1/4}-\frac{33}{128}n^{-3/4}+O\left(n^{-5/4}\right)$$
    $$\left(f(n)+\frac{1}{4}\right)^{1/2}-\frac{1}{2}=n^{1/4}-\frac{1}{2}+\frac{1}{8}n^{-1/4}-\frac{1}{128}n^{-3/4}+O\left(n^{-5/4}\right)$$
    et l'inégalité de dessus donne donc pas mieux que
    $$a_n=n^{1/4}-\frac{1}{2}+\frac{1}{8}n^{-1/4}+O\left(n^{-3/4}\right)$$
  • Je crois que si $0<a_{1}\leq\frac{-1+\sqrt{4f(1)+1}}{2}$ alors $f(n)$ croissante implique que $a_{n}$ est croissante dès $n=1$.

    Ajout : reprenons $f(n)=n^{1/2}$ et $a_{1}=\frac{-1+\sqrt{5}}{2}$ alors, comme $a_{n}$ est croissante, on peut appliquer le résultat précédant et
    $$
    a_{n}=n^{1/4}-\frac{1}{2}+\frac{1}{8}n^{-1/4}+O\big(n^{-3/4}\big).

    $$ Paramétrons la suite en notant $a_{r,1}=r>0$ et $a_{r,n+1}=\frac{f(n)}{1+a_{r,n}}$ avec $f>0$ croissante. Alors pour tout $r\neq r'$, on a $a_{r,n}-a_{r',n}$ qui est une suite alternée vérifiant $a_{r,n}-a_{r',n}=o(1)$. En appliquant ça à $f(n)=n^{1/2}$ et en prenant $r=\frac{-1+\sqrt{5}}{2}$ et $r'=1$ on a la réponse à la question initiale (pas plus de 2 termes dans le développement)
    $$
    a_{1,n}=n^{1/4}-\frac{1}{2}+o(1).

    $$ C'est un peu lourd et en plus je dois vérifier un truc pour être net sur $a_{r,n}-a_{r',n}=o(1)$. Etanche c'est quoi la source de cet exo ?
  • Pour en revenir aux conjectures de Chaurien, on a $a_n>b_n$ mais $a_n<b_{n+1}$ pour $5\le n\le 3000$ (j'ai peut-être fait une erreur d'indice) et la suite $(a_n)$ semble en effet croissante.
  • Je propose en fait de raisonner par récurrence sans se soucier de la croissance (le truc est d'incrémenter ici un cran plus loin à droite).
    Si $a_{1}=1,\ a_{n+1}=\frac{n^{1/2}}{1+a_{n}}$ montrons que pour $n\geq 4$
    $$
    \left((n-1)^{1/2}+\frac{1}{4}\right)^{1/2}-\frac{1}{2}<a_{n}<\left((n+1)^{1/2}+\frac{1}{4}\right)^{1/2}-\frac{1}{2}.

    $$Vrai pour $n=4$. Si vrai au rang $n>4$.
    \begin{align*}
    \left(n^{1/2}+\frac{1}{4}\right)^{1/2}-\frac{1}{2}&<a_{n+1}<\left((n+2)^{1/2}+\frac{1}{4}\right)^{1/2}-\frac{1}{2} \\

    &\Leftrightarrow
    \left(n^{1/2}+\frac{1}{4}\right)^{1/2}-\frac{1}{2}<\frac{n^{1/2}}{1+a_{n}}<\left((n+2)^{1/2}+\frac{1}{4}\right)^{1/2}-\frac{1}{2} \\

    &\Leftrightarrow
    \frac{n^{1/2}}{\left((n+2)^{1/2}+\frac{1}{4}\right)^{1/2}-\frac{1}{2}}-1<a_{n}<\frac{n^{1/2}}{\left(n^{1/2}+\frac{1}{4}\right)^{1/2}-\frac{1}{2}}-1.\\

    \text{Et on a bien pour $n\geq4$}\\

    \left((n-1)^{1/2}+\frac{1}{4}\right)^{1/2}-\frac{1}{2}&<\frac{n^{1/2}}{\left((n+2)^{1/2}+\frac{1}{4}\right)^{1/2}-\frac{1}{2}}-1
    \qquad \text{et} \\

    \frac{n^{1/2}}{\left(n^{1/2}+\frac{1}{4}\right)^{1/2}-\frac{1}{2}}-1&<\left((n+1)^{1/2}+\frac{1}{4}\right)^{1/2}-\frac{1}{2}

    \end{align*} dont la vérification est laissée au lecteur (:P) Bon même si je me suis emmêlé les pinceaux je pense que la solution est par là...
    De l'inégalité de départ on tire donc
    $$a_n=n^{1/4}-\frac{1}{2}+\frac{1}{8}n^{-1/4}+O\left(n^{-3/4}\right).$$
  • Bon allez démonstration de l'inégalité la plus simple
    $$\frac{n^{1/2}}{\left(n^{1/2}+\frac{1}{4}\right)^{1/2}-\frac{1}{2}}-1<\left((n+1)^{1/2}+\frac{1}{4}\right)^{1/2}-\frac{1}{2}
    $$ en fait pour $n>0$. On va montrer que pour tout réel $x>0$ on a (1)
    $$\frac{x}{\sqrt{x+\frac{1}{4}}-\frac{1}{2}}-\frac{1}{2}<\left(\sqrt{x^{2}+1}+\frac{1}{4}\right)^{1/2}
    $$ Posons $y=\sqrt{x+\frac{1}{4}}-\frac{1}{2}$ de sorte qu'aussi $y>0$. Comme $x=y^{2}+y$ l'inégalité (1) devient
    $$y+\frac{1}{2}<\left(\sqrt{y^{4}+2y^{3}+y^{2}+1}+\frac{1}{4}\right)^{1/2}
    $$ Élevons au carré
    $$y^{2}+y<\sqrt{y^{4}+2y^{3}+y^{2}+1},
    $$ encore une fois
    $$y^{4}+2y^{3}+y^{2}<y^{4}+2y^{3}+y^{2}+1
    $$ ce qui n'est pas faux.
  • L'inégalité la moins simple est fausse...Donc comme ça ça ne marche pas.
  • Mon dernier essai en espérant que les pinceaux soient mieux rangés. Tout découle de la première indication de Chaurien confirmée par Mathcoss.
    Si $a_{1}=1$ et $a_{n+1}=\frac{n^{1/2}}{1+a_{n}}$ montrons par récurrence que pour $n\geq5$
    $$\left((n-1)^{1/2}+1/4\right)^{1/2}-1/2\leq a_{n}\leq\left(n^{1/2}+1/4\right)^{1/2}-1/2
    $$ Vrai pour $n=5$. Ensuite l'hypothèse de récurrence et la formule pour $a_{n+1}$ donne
    $$\frac{n^{1/2}}{\left(n^{1/2}+1/4\right)^{1/2}+1/2}\leq a_{n+1}\leq\frac{n^{1/2}}{\left((n-1)^{1/2}+1/4\right)^{1/2}+1/2}.
    $$ Et on a bien d'un côté trivialement
    $$\left(n^{1/2}+1/4\right)^{1/2}-1/2\leq\frac{n^{1/2}}{\left(n^{1/2}+1/4\right)^{1/2}+1/2},
    $$ et de l'autre pour $n\geq5$ l'étude de fonction montre que
    $$\frac{n^{1/2}}{\left((n-1)^{1/2}+1/4\right)^{1/2}+1/2}\leq\left((n+1)^{1/2}+1/4\right)^{1/2}-1/2.
    $$ On a sauf erreur le meilleur développement possible pour $a_{n}$ comme annoncé
    $$a_{n}=n^{1/4}-1/2+\frac{1}{8}n^{-1/4}+O\left(n^{-3/4}\right).
    $$ Désolé si je me suis encore fourvoyé.
  • Math Coss a écrit:
    Pour la première, je parierais pour
    \[ u_n=n^{1/4}-\frac12+\frac18n^{-1/4}+\frac18n^{-3/4}+o(n^{-3/4})
    \] Le dernier terme n'est pas très étayé

    C'est effectivement faux pour le dernier terme. En admettant que $a_n$ ait un développement asymptotique en les puissances (inverses) de $n^{1/4}$, on trouve les coefficients en injectant ce développement dans $a_{n+1}$ et dans $\dfrac{\sqrt{n}}{1+a_n}$ et en comparant les deux.
    Avec maple, je trouve :
    \[ a_n = n^{1/4} - \dfrac{1}{2} + \dfrac{1}{8 n^{1/4}} - \dfrac{17}{128 n^{3/4}} - \dfrac{1}{16 n} + \dfrac{17}{1024n^{5/4}} + o\left(\dfrac{1}{n^{5/4}}\right) .
    \] On pourrait continuer et pousser le développement arbitrairement loin (toujours en admettant son existence).
  • Sinon, pour la deuxième question, il semblerait qu'on a toujours $a_n \sim n^{\alpha/2}$. Pour le terme d'après, lorsque $\alpha < 2$, on a apparemment $a_n = n^{\alpha/2} - \dfrac{1}{2} + o(1)$. Par contre, pour $\alpha\geqslant 2$, ça a l'air plus compliqué.
  • Bien vu Guego. Pour alpha=2.5 par exemple il semble que a_n/n^1.25 tend vers 1 en oscillant...
  • J'ai une erreur de signe et une confusion entre 16/128 et 17/128.
Connectez-vous ou Inscrivez-vous pour répondre.